2005 AMC 10B Problems/Problem 5

Revision as of 12:17, 27 August 2008 by Abacadaea (talk | contribs) (Problem)

This is an empty template page which needs to be filled. You can help us out by finding the needed content and editing it in. Thanks.

Problem

Brianna is using part of the money she earned on her weekend job to buy several equally priced CDs. She used one fifth of the money to buy one third of the CD's. What fraction of the money will she have left once she buys the CD's?

$\mathrm{(A)}\ {{{\frac{1} {5}}}} \qquad \mathrm{(B)}\ {{{\frac{1} {3}}}} \qquad \mathrm{(C)}\ {{{\frac{2} {5}}}} \qquad \mathrm{(D)}\ {{{\frac{2} {3}}} \qquad \mathrm{(E)}\ {{{\frac{4} {5}}}}$ (Error compiling LaTeX. Unknown error_msg)

Solution

This problem needs a solution. If you have a solution for it, please help us out by adding it.

See Also